LSAT and Law School Admissions Forum

Get expert LSAT preparation and law school admissions advice from PowerScore Test Preparation.

 Administrator
PowerScore Staff
  • PowerScore Staff
  • Posts: 8919
  • Joined: Feb 02, 2011
|
#81329
Complete Question Explanation

Parallel Reasoning. The correct answer choice is (E).

Answer choice (A):

Answer choice (B):

Answer choice (C):

Answer choice (D):

Answer choice (E): This is the correct answer choice.

This explanation is still in progress. Please post any questions below!
 jlam061695
  • Posts: 62
  • Joined: Sep 17, 2016
|
#30526
I chose C initially for this question, and then I went back and picked B. Why are they incorrect? And I don't understand why E is correct?

The flaw in C matches closely with that of the stimulus: though x is harmful in large/larger quantities, it does not prove that a smaller quantity of x harms y. Perhaps the reason why it is incorrect was diction-related? Even though I felt like the flaw matched, I also felt that the phrase "it proves only" was too strong (which was not present in the stimulus). I have trouble with some flaw questions because the correct answer usually does not have language that matches up perfectly with the stimulus.
 Emily Haney-Caron
PowerScore Staff
  • PowerScore Staff
  • Posts: 577
  • Joined: Jan 12, 2012
|
#30607
Hi jlam,

Great question. Thanks for explaining what your reasoning was on this one.

Here's the abstraction of the stimulus:
Just because a thing is bad in large quantities doesn't mean it is bad in small quantities. Proof of this is that another thing in large quantities is harmful but is helpful in small quantities.

You want the answer choice that matches this. E comes closest; substitute tobacco smoke for watching TV, and vitamin A for sleep, and it's similar enough.

C and B don't do this at all.
B can be abstracted as: Just because you should have a certain amount of something doesn't mean larger amounts of the thing are better. Too much of that same thing can cause problems.
Here, there's nothing standing in for vitamin A, and also the reasoning doesn't match up.

C can be abstracted as: Just because a thing is bad in large quantities doesn't mean it is bad in small quantities. (good so far!!) That only proves you should make sure you're using the right amount based on various factors.
The first part is perfect. The second part, though, totally differs from the stimulus. What about the second thing (vitamin A)? What about giving an example to support the logic of the first sentence? Nope. Doesn't work.

I hope that helps!
 jlam061695
  • Posts: 62
  • Joined: Sep 17, 2016
|
#30622
Thanks for your explanation, Emily, that cleared it up for me! :)
 ksandberg
  • Posts: 21
  • Joined: Sep 03, 2016
|
#36180
Hello,

I would like to make sure my thought process is correct as to why A is incorrect. I originally chose A, but I think it is wrong because we are not discussing something that is harmful in large quantities, but helpful in small quantities. Rather, we are discussing something that is helpful in large quantities and may be less helpful in small quantities? Is this correct? Thank you for your time.
 Francis O'Rourke
PowerScore Staff
  • PowerScore Staff
  • Posts: 471
  • Joined: Mar 10, 2017
|
#36208
Hi Sandberg,

There are a few elements that you are missing in your analysis of answer choice (A). I'm not sure though if bleach turning fabric white should be thought of as harmful or helpful. The stimulus and answer choice (E) clearly gave us examples of bad effects of an action, countered by helpful effects of small amounts of an analogous action.

Choice (A) seems to give us a neutral effect of an action and then states that the effect may not occur at smaller concentrations. I want a clearly negative effect of an action (whitening can be good or bad depending on what you bleach) followed by a clearly positive effect of a separate, analogous action.
 PB410
  • Posts: 39
  • Joined: Apr 01, 2017
|
#36795
Hi, i'm trying to recognize flaws as I go through each stimulus. I need some help identifying the flaw present, if there is one. So far I have as the conclusion in the first sentence and the support in the second. The conclusion relies on countering a flaw to reach a conclusion by stating, that the fact that tobacco smoke inhaled by smokers harms smokers, does not prove that a smaller amount of smoke hurts non nonsmokers. Would the flaw being countered fall under an Error in Assessing the Force of Evidence, or would it be responding to an Error in Division?
And secondly, the stimulus as a whole, in relying on the level of toxicity and quantity of other substances as support to justify the conclusion, would that be a General Lack of Relevant Evidence for a Conclusion? The author makes a leap in comparing the effects of Vitamins and other substances with the effects of smokers.
 nicholaspavic
PowerScore Staff
  • PowerScore Staff
  • Posts: 271
  • Joined: Jun 12, 2017
|
#36851
Hi PB410,

This is a great question!

You are correct that there is (much) flawed reasoning in this stimulus. ;)

With respect to your first question regarding "Error in Assessing the Force of Evidence" or "Error in Division," I would actually go with a third alternative identified in our "Errors in Use in Evidence." Specifically, I would say the error made is that the lack of evidence for a position (that second-hand smoke is bad for you instead of smoke directly inhaled) is taken to prove that position is true. Remember the lack of evidence against a position does not undeniably prove a position!

With respect to your second question about what the overall flaw is, you are correct! It is a General Lack of Relevant Evidence for a Conclusion because the argument fails to provide any relevant information to support its conclusion. Well done!

Hope this helped!

Get the most out of your LSAT Prep Plus subscription.

Analyze and track your performance with our Testing and Analytics Package.